Tải bản đầy đủ (.pdf) (70 trang)

TUYỂN TẬP CÁC BÀI TOÁN TRONG KÌ THI CHỌN ĐỘI TUYỂN QUỐC GIA TỪ NĂM 1990 ĐẾN NĂM 2012

Bạn đang xem bản rút gọn của tài liệu. Xem và tải ngay bản đầy đủ của tài liệu tại đây (1.09 MB, 70 trang )

1

TUYỂN TẬP CÁC BÀI TOÁN
TRONG KÌ THI CHỌN ĐỘI TUYỂN QUỐC GIA
TỪ NĂM 1990 ĐẾN NĂM 2012
**********
PHẦN 1. ĐỀ BÀI
Bài 1. (TST 1990, bài 6)
Cho n học sinh
( 3)
n

đứng thành một vòng tròn và quay mặt vào cô giáo ở tâm đường
tròn. Mỗi lần cô giáo thổi còi thì hai em nào đó đứng cạnh nhau đổi chỗ cho nhau, các
em còn lại đứng im. Tìm số M nhỏ nhất sao cho sau M lần thổi còi, bằng cách đổi chỗ
như trên một cách thích hợp thì các em học sinh đứng được thành một vòng tròn mới
sao cho: Hai em lúc đầu đứng cạnh nhau, nhưng hai em đó, tạm gọi là A và B, thì nếu
lúc đầu A đứng bên tay trái của B thì lúc kết thúc, A đứng bên tay phải của B.
Bài 2. (TST 1991, bài 5)
Với mỗi số tự nhiên
n
, ta định nghĩa số
f n
( )
như sau
f
(1) 1

và khi
n
1



thì
k k
f n a p a p a p
1 1 2 2
( ) 1      ,
trong đó
k
a
a a
k
n p p p
1 2
1 2

 là phân tích thành thừa số nguyên tố của
n
(các số nguyên tố
k
p p p p
1 2 3
, , , ,
là đôi một khác nhau và
k
a a a a
1 2 3
, , , ,
là nguyên dương). Với mỗi số
tự nhiên
k

, ta đặt




k
f n f f f n
( ) ( )

, trong đó ở vế phải có đúng
k
lần chữ f .
Chứng minh rằng với số tự nhiên
a
cho trước, tồn tại số tự nhiên
k
0
để với mọi số
nguyên
k k
0

thì tổng
k k
f a f a
1
( ) ( )

 không phụ thuộc vào
k

.
Bài 3. (TST 1992, bài 6)
Trong một hội thảo khoa học, tất cả các đại biểu biết
n
2
ngôn ngữ,
n
2.

Mỗi người biết
đúng 2 ngôn ngữ và hai đại biểu bất kì biết chung nhiều nhất 1 ngôn ngữ. Biết rằng một
số nguyên
k
thỏa mãn
k n
1 1
  
đều có không quá
k
1

ngôn ngữ mà mỗi ngôn ngữ
này đều có không quá
k
người biết. Chứng minh rằng ta có thể chọn ra một nhóm
n
2

đại biểu biết tổng cộng
n

2
ngôn ngữ và mỗi ngôn ngữ có đúng 2 người trong đó biết.
2

Bài 4. (TST 1993, bài 6)
Xét
n
điểm
n
A A A
1 2
, , ,
với
n
2

trong không gian, trong đó không có 4 điểm nào
đồng phẳng. Mỗi cặp điểm
i j
A A
,
với
i j

được nối với nhau bởi một đoạn thẳng.Tìm
giá trị lớn nhất của
n
sao cho có thể tô tất cả các đoạn thẳng đó bởi hai màu xanh hoặc
đỏ thỏa mãn đồng thời các điều kiện sau:
1) Mỗi đoạn thẳng được tô bằng đúng một màu.

2) Với mỗi
i n
1,2,3, ,

thì số đoạn thẳng có một đầu mút là
i
A
được tô màu xanh
không vượt quá 4.
3) Với mỗi đoạn thẳng
i j
A A
được tô màu đỏ đều tìm được ít nhất một điểm
k
A
với
k

khác
i j
,
sao cho các đoạn thẳng
i k j k
A A A A
, được tô màu xanh.
Bài 5. (TST 1994, bài 2)
Xét phương trình nghiệm nguyên dương
x y z t Nxyzt N
2 2 2 2
0

     

với
N
là số nguyên dương cho trước.
1) Chứng minh rằng tồn tại vô số giá trị
N
sao cho phương trình này có nghiệm
nguyên dương (mỗi nghiệm là một bộ bốn số nguyên dương
x y z t
, , ,
).
2) Chứng minh rằng với
k
N m
4 (8 7)
 
với
k m
,
là các số không âm thì phương trình
này không có nghiệm nguyên dương.
Bài 6. (TST 1995, bài 6)
Cho hàm số thực
x
f x x
x
3
2
2 3

( ) ,
3( 1)

 


.
1. Chứng minh rằng tồn tại hàm số
g x
( )
liên tục trên

sao cho
f g x x
( ( ))


g x x
( )


với mọi số thực
x
.
2. Chứng minh rằng tồn tại số thực
a
1

để dãy
n

u n
( ), 1,2,3,
 được xác định bởi
n n
u a u f u n
0 1
, ( ), 1,2,3,

   tuần hoàn với chu kỳ dương nhỏ nhất bằng 1995.
3

Bài 7. (TST 1996, bài 3)
Xét các số thực
a b c
, , .
Tìm giá trị nhỏ nhất của biểu thức
f a b c a b b c c a a b c
4 4 4 4 4 4
4
( , , ) ( ) ( ) ( ) ( )
7
         .
Bài 8. (TST 1997, bài 3)
Tìm số thực
k
lớn nhất sao cho tồn tại vô hạn số tự nhiên
n
a n
( ), 1


thỏa mãn đồng thời
1)
n
n
a
1997
 với mọi
n
.

2) Với mỗi
n
2,

ta đều có
k
n n
u a

với
n
u
là ước chung lớn nhất của tất cả các số có dạng
i k
a a


i k n
.
 


Bài 9. (TST 1998, bài 5)
Giả sử
d
là ước dương của
1998
5 1998

. Chứng minh rằng
d
có thể biểu diễn được dưới
dạng
d x xy y
2 2
2 2 3
   với
x y
,
là các số nguyên dương khi và chỉ khi
d
chia 20 có số
dư là 3 hoặc 7.
Bài 10. (TST 1999, bài 1)
Cho một số nguyên tố lẻ
p
thỏa mãn
h
p
2 1(mod )
 với mọi h p h

*
1,
  

và một số
chẵn
p
a p
;
2
 

 
 
. Xét dãy số
n
a
( )
xác định bởi
n n
a a a p b n
0 1
, , 0,1,2,

    với
n
b
là ước số lẻ lớn nhất của
n
a

.
Chứng minh rằng
n
a
( )
là dãy số tuần hoàn và tìm chu kì dương nhỏ nhất của nó.
Bài 11. (TST 2000, bài 4)
Cho ba số nguyên dương
a b c
, ,
đôi một nguyên tố cùng nhau. Số nguyên dương n được
gọi là “số bướng bỉnh” nếu như n không biểu diễn được dưới dạng
n abx bcy caz
  

trong đó
x y z
, ,
là các số nguyên dương. Hỏi có tất cả bao nhiêu số bướng bỉnh?
Bài 12. (TST 2001, bài 5)
Cho số nguyên dương
n
lớn hơn 1. Trong không gian vuông góc
Oxyz
, gọi T là tập hợp
tất cả các điểm có tọa độ là
x y z
( , , )
với
x y z

, ,
là các số nguyên dương thỏa
x y z n
1 , ,
 
.
Tô màu tất cả các điểm thuộc tập hợp T sao cho: nếu điểm
A x y z
0 0 0
( , , )
được tô màu thì
những điểm có dạng
B x y z
1 1 1
( , , )
với
x x y y z z
1 0 1 0 1 0
, ,
  
sẽ không được tô màu.
Tìm giá trị lớn nhất các điểm được tô màu thỏa mãn điều kiện trên.
4

Bài 13. (TST 2002, bài 5)
Hãy tìm tất cả các đa thức
P x
( )
với hệ số nguyên sao cho đa thức sau
Q x x x P x

2 2
( ) ( 6 10) ( ) 1
   

là bình phương của một đa thức với hệ số nguyên.
Bài 14. (TST 2003, bài 3)
Cho hàm số
f
:
 
  
thỏa mãn đồng thời các điều kiện sau:
i) f f n
2003
(0,0) 5 , (0, ) 0
 
với mọi n là số nguyên khác 0.
ii)
f m n f m n f m n
f m n f m n
( 1, ) ( 1, 1) ( 1, 1)
( , ) ( 1, ) 2
2 2 2
          
    
     
     

với mọi số nguyên dương m và mọi số nguyên n.
Chứng minh rằng tồn tại số nguyên dương M sao cho

f M n
( , ) 1

với mọi số nguyên n
thỏa n
2003
5 1
2

 và
f M n
( , ) 0

với mọi số nguyên n thỏa n
2003
5 1
2

 .
Bài 15. (TST 2004, bài 6)
Cho S là một tập hợp gồm một số số nguyên dương mà số nhỏ nhất và số lớn nhất
trong S là hai số nguyên tố cùng nhau. Với mỗi số tự nhiên n, ký hiệu
n
S
là tập hợp
gồm tất cả các số tự nhiên mà mỗi số đều là tổng của nhiều nhất
n
số (không nhất thiết
đôi một khác nhau) thuộc tập S. Quy ước 0 là tổng của 0 số thuộc S. Gọi a là số lớn nhất
trong tập S. Chứng minh rằng tồn tại số nguyên dương k và số nguyên b sao cho

n
S an b
 
với mọi
n k

.
Bài 16. (TST 2005, bài 6)
Một số nguyên dương được gọi là “số kim cương 2005” nếu trong biểu diễn thập phân
của nó có 2005 số 9 đứng cạnh nhau liên tiếp. Dãy


n
a n
, 1,2,3,
 là dãy tăng ngặt các
số nguyên dương thỏa mãn
n
a nC
 (C là hằng số thực dương nào đó).
Chứng minh rằng dãy số


n
a n
, 1,2,3,
 chứa vô hạn “số kim cương 2005”.
Bài 17. (TST 2005 bổ sung, bài 3)
Cho bảng vuông kích thước
n n n

, 2
 
. Hai ô vuông được gọi là kề nhau nếu chúng là
hai ô liên tiếp trên cùng một hàng hoặc cùng một cột .
5

1. Tìm số nguyên dương
p
lớn nhất để tồn tại một cách đánh dấu
p
ô trên bảng thoả
mãn: Trong số các ô kề với mỗi ô được đánh dấu có không quá một ô được đánh dấu.
2. Hai cách đánh dấu được gọi là
q
kề nhau nếu chúng đều có
q
ô được đánh dấu,
trong đó có
q
1

ô có vị trí trùng nhau trên bảng và hai ô còn lại của hai cách này là hai
ô kề nhau. Tìm số nguyên dương
q
nhỏ nhất để tồn tại một cách đánh dấu
q
ô trên
bảng thoả các điều kiện:
i) Tất cả các ô kề với mỗi ô được đánh dấu đều không được đánh dấu.
ii) Mọi cách đánh dấu

q
kề nhau với nó đều không thoả mãn điều kiện i).
Bài 18. (TST 2006, bài 3)
Trong không gian cho 2006 điểm mà trong đó không có 4 điểm nào đồng phẳng. Người
ta nối tất cả các điểm đó lại bởi các đoạn thẳng. Số nguyên dương
m
được gọi là số tốt
nếu ta có thể gán cho mỗi đoạn thẳng trong các đoạn thẳng đã nối bởi một số nguyên
dương không vượt quá m sao cho mỗi tam giác tạo bởi ba điểm bất kì trong số các điểm
đó đều có hai cạnh được gán bởi hai số bằng nhau và cạnh còn lại gán bởi số lớn hơn
hai số đó. Tìm số tốt có giá trị nhỏ nhất.
Bài 19. (TST 2007, bài 6)
Cho đa giác 9 cạnh đều
H
( )
. Xét ba tam giác với các đỉnh là các đỉnh của đa giác (H) đã
cho sao cho không có hai tam giác nào có chung đỉnh. Chứng minh rằng có thể chọn
được từ mỗi tam giác 1 cạnh sao cho 3 cạnh này bằng nhau.
Bài 20. (TST 2008, bài 5)
Cho tam giác ABC nhọn, không cân có O là tâm đường tròn ngoại tiếp. Gọi
AD BE CF
, ,

lần lượt là các đường phân giác trong của tam giác. Trên các đường thẳng
AD BE CF
, ,

lần lượt lấy các điểm
L M N
, ,

sao cho
AL BM CN
k
AD BE CF
  
(k là hằng số dương).
Gọi
O O O
1 2 3
( ),( ),( )
lần lượt là các đường tròn đi qua L, tiếp xúc với OA tại A; đi qua M
tiếp xúc với OB tại B và đi qua N tiếp xúc với OC tại C.
1) Chứng minh rằng với k
1
2

, ba đường tròn
O O O
1 2 3
( ),( ),( )
có đúng hai điểm chung
và đường thẳng nối hai điểm đó đi qua trọng tâm tam giác ABC.
6

2) Tìm tất cả các giá trị k sao cho 3 đường tròn
O O O
1 2 3
( ),( ),( )
có đúng hai điểm chung.
Bài 21. (TST 2009, bài 5)

Cho đường tròn
O
( )
có đường kính AB và M là một điểm bất kì nằm trong
O
( )
, M
không nằm trên AB. Gọi N là giao điểm của phân giác trong góc M của tam giác AMB
với đường tròn
O
( )
. Đường phân giác ngoài góc AMB cắt các đường thẳng NA, NB lần
lượt tại
P Q
,
. Đường thẳng MA cắt đường tròn đường kính NQ tại R, đường thẳng MB
cắt đường tròn đường kính NP tại S và
R S
,
khác M. Chứng minh rằng đường trung
tuyến ứng với đỉnh N của tam giác NRS luôn đi qua một điểm cố định khi M di động
phía trong đường tròn.
Bài 22. (TST 2010, bài 3)
Gọi một hình chữ nhật có kích thước
1 2

là hình chữ nhật đơn và một hình chữ nhật có
kích thước
2 3


, bỏ đi 2 ô ở góc chéo nhau (tức có có 4 ô vuông con) là hình chữ nhật
kép. Người ta ghép khít các hình chữ nhật đơn và hình chữ nhật kép này lại với nhau
được một bảng hình chữ nhật có kích thước là
2008 2010

. Tìm số nhỏ nhất các hình
chữ nhật đơn có thể dùng để ghép.
Bài 23. (TST 2011, bài 3)
Cho cho n nguyên dương thỏa
n
3

và n số thực
n
x x x x
1 2 3
, , , ,
thỏa mãn đồng thời
(i)
n
x x x x
1 2 3
0
    
.
(ii)
n
x x x x n n
2 2 3 2
1 2 2

( 1)
     
.
(iii)
n
x x x x
1 2 3

   
.
Tìm giá trị lớn nhất và nhỏ nhất của
f x x
1 2
 
.
Bài 24. (TST 2012, bài 2)
Trên một cánh đồng hình chữ nhật kích thước
m n

ô vuông gồm m hàng và n cột,
người ta đặt một số máy bơm nước vào các ô vuông. Biết rằng mỗi máy bơm nước có
thể tưới nước không những cho ô vuông chứa nó và các ô vuông có chung cạnh với ô
đó mà còn có thể tưới cho các ô vuông cùng cột với nó và cách nó đúng một ô vuông.
Tìm số nhỏ nhất các máy bơm nước cần đặt để các máy bơm đó có thể tưới hết cả cánh
đồng trong hai trường hợp:
1)
m
4

.

2)
m
3

.
7

PHẦN 2. LỜI GIẢI CHI TIẾT
Bài 1.
Cho n học sinh
( 3)
n

đứng thành một vòng tròn và quay mặt vào cô giáo ở tâm đường
tròn. Mỗi lần cô giáo thổi còi thì hai em nào đó đứng cạnh nhau đổi chỗ cho nhau, các
em còn lại đứng im. Tìm số M nhỏ nhất sao cho sau M lần thổi còi, bằng cách đổi chỗ
như trên một cách thích hợp thì các em học sinh đứng được thành một vòng tròn mới
sao cho: Hai em lúc đầu đứng cạnh nhau, nhưng hai em đó, tạm gọi là A và B, thì nếu
lúc đầu A đứng bên tay trái của B thì lúc kết thúc, A đứng bên tay phải của B.
Lời giải.
Ta thấy rằng yêu cầu của bài toán có thể phát biểu thành:
Giả sử ban đầu
n
bạn học sinh đứng theo một thứ tự nhất định nào đó trên vòng tròn
theo cùng kim đồng hồ. Tìm số lần đổi chỗ nhỏ nhất sao cho các học sinh vẫn đứng theo
thứ tự đó trên vòng tròn nhưng ngược chiều kim đồng hồ.
Đánh số các học sinh này là
n n
1,2,3, , 2, 1
 

theo chiều kim đồng hồ và ở trạng thái
cuối, các học sinh cũng được đánh số như thế nhưng ngược chiều kim đồng hồ.
Ta thấy rằng với ba học sinh
A B C
, ,
đứng theo thứ tự đó thì khi đổi chỗ B và C, ta nhận
được thứ tự mới là
A C B
, ,
. Ta có thể coi trong việc đổi chỗ đó, B đứng yên và C chuyển
qua vị trí xen giữa A và B. Như thế, với bất kì dãy các cách đổi chỗ nào, ta luôn có thể
chọn ra được ít nhất 1 học sinh đứng yên trong suốt quá trình đó, giả sử là học sinh
được đánh số 1. Rõ ràng cũng không thể có trường hợp có ba học sinh
A B C
, ,
nào đó
(không nhất thiết đứng cạnh nhau) đứng yên vì khi đó, các học sinh này sẽ xác định
một chiều không đổi trong suốt quá trình đổi chỗ và như thế thì không thỏa mãn đề bài.
Ta sẽ chứng minh hai nhận xét sau:
Nhận xét 1.
Với
n
3

, ta luôn có thể chọn hai học sinh đứng yên trong quá trình đổi chỗ và việc
chọn này không ảnh hưởng đến số lần đổi chỗ nhỏ nhất. Điều này có nghĩa là nếu trong
cách chọn hai học sinh này, có một cách đổi chỗ thỏa mãn đề bài cần
x
lần thì trong
một cách chọn hai học sinh khác, cũng tồn tại một cách đổi chỗ với

x
lần như thế.
8

Thật vậy,
Ta cố định một học sinh
A
tại mọi thời điểm. Xét một cách chọn cặp học sinh với hai
học sinh được chọn là
A

B
. Giả sử đoạn thẳng
AB
chia
n
2

học sinh còn lại thành
2 phần
X Y
1 1
,
, trong đó
X
1
nằm trên cung tròn hướng từ
B
đến
A

,
X
2
nằm trên cung
còn lại. Giả sử
X a Y b
1 1
,
 
với
a b n
2 1.
   

A
b
1
b
2
C
B
a
1
a
2

Dễ thấy rằng các học sinh trong tập hợp
X
1
phải di chuyển vượt qua dây

AB
để đi về
phía đối diện với
f
1
lần đổi chỗ và các học sinh
X
2
phải di chuyển vượt qua dây
AB
về
phía đối diện với
f
2
lần đổi chỗ, tổng số lần đổi chỗ là
f f
1 2

lần.
Giả sử trong một cách di chuyển nào đó, ở tập hợp
X
1
,

a
1
học sinh di chuyển về phía
A

b

1
học sinh di chuyển về phía
B
.
Tương tự, trong tập hợp
X
2
, số học sinh di
chuyển về phía
A B
,
lần lượt là
a b
2 2
, .

Do
n
2 0
 
nên tồn tại học sinh thuộc một trong hai tập hợp
X X
1 2
,
; ta giả sử
C
nằm
kề với
B
và thuộc về

X
2
.
Khi đó, nếu chuyển hai học sinh cố định từ
A B
,
thành
A C
,

thì hai tập hợp
X Y
1 1
,
sẽ trở thành
X Y
2 2
,
với X X Y Y
2 1 2 1
1, 1
   
.
Ta sẽ chỉ ra một cách di chuyển tương ứng bằng với số cách di chuyển cần
f f
1 2

lần
đổi chỗ như trên.
Ta sẽ giữ nguyên các học sinh di chuyển về phía

A
ở hai bên. Số học sinh di chuyển về
phía ngược lại ở tập
X
2
lúc này có thêm
B
hay b b
2 2
1

 
, số học sinh di chuyển về phía
ngược lại ở tập
Y
2
lúc này mất đi
C
hay c c
2 2
1.

 

9

Ta vẫn cho các học sinh của tập
X
2
di chuyển vượt qua

C
(thay vì chỉ vượt qua
B
như
trước), mỗi học sinh tốn thêm một lần đổi chỗ nên tổng cộng mất thêm b
2
1

lần. Tiếp
theo, cho các học sinh của tập
Y
2
di chuyển vượt qua
C
về phía bên kia. Các học sinh
cũ cũng tốn các lần đổi chỗ giống như cũ (đều phải vượt qua tất cả học sinh từ tập
X
2

mới chuyển qua), tuy nhiên, do chỉ còn c
2
1

học sinh nên sẽ không bị tốn thêm b
2
1


số lần đổi chỗ tương ứng.
Do đó, số lần đổi chỗ tăng lên và giảm đi một lượng như nhau nên chúng bằng nhau. Vì

hai cách chọn học sinh
B C
,
kề nhau như thế đều có các số lần đổi chỗ tương ứng giống
nhau nên với hai cách chọn học sinh bất kì, các số lần đổi chỗ của chúng cũng sẽ tương
ứng giống nhau. Nhận xét được chứng minh.
Nhận xét 2.
Nếu gọi
f n
( )
là số lần chuyển chỗ nhỏ nhất trong trường hợp có
n
học sinh, ta có
f f f n f n n n
(3) 1, (4) 2, ( ) ( 2) 2, 5
      
.
Thật vậy, nếu có 3 học sinh thì ta chỉ cần đổi chỗ một cặp học sinh và trong trường hợp
có 4 học sinh thì đổi chỗ hai cặp kề nhau thì có cách sắp xếp thỏa mãn đề bài. Dễ dàng
suy ra
f f
(3) 1, (4) 2.
 

Xét
n
5

tùy ý, với
n

2

học sinh, ta có số lần chuyển chỗ ít nhất là
f n
( 2)

. Ta thêm
hai học sinh thứ
n
1


n
vào vòng tròn và theo nhận xét 1, ta có thể chọn hai học sinh
này cố định. Các học sinh còn lại phải di chuyển vào giữa hai học sinh cố định này. Rõ
ràng trong cách đổi chỗ nhỏ nhất đã thực hiện với
n
2

học sinh còn lại, nếu muốn
chuyển chỗ cho
n
học sinh này thì các học sinh trước đó đều phải vượt qua thêm một
trong hai học sinh cố định đã nêu và đòi hỏi cần thêm
n
2

lần đổi chỗ nữa bắt buộc
nữa. Theo nhận xét 1 thì cách chuyển này vẫn là nhỏ nhất và đúng bằng
f n

( )
, suy ra
f n f n n
( ) ( 2) 2
   
với
n
5.

Từ đó suy ra
f k k k k
(2 ) 2 4 6 2 2 ( 1)
       

f k k k
2
(2 1) 1 3 5 2 1
       
.
Ta sẽ chỉ ra một cách đổi chỗ thỏa mãn đề bài.
- Với
n
chẵn, đặt
n k
2 ,

ta cố định học sinh 1 và
n
.
Xét các học sinh

n
n n
1, 2, 1
2
  
di chuyển về phía học sinh
n
lần lượt theo thứ tự đó ;
các học sinh còn lại là
n
2,3,4, ,
2
di chuyển về phía học sinh 1 lần lượt theo thứ tự đó
10

để vào khoảng giữa hai học sinh này. Dễ thấy rằng: học sinh thứ
i

n i
1
 
với
n
i
2,3,4, ,
2
 đều cần số lần đổi chỗ là
i
1


. Do đó, tổng số lần đổi chỗ của các học
sinh này là
n n n
k k
2 1 2 3 1 1 ( 1)
2 2 2
   
        
   
   
.
- Với
n
lẻ, đặt
n k
2 1,
 
tương tự, ta cũng cố định học sinh
1

n
.

Xét các học sinh
n
n n
3
1, 2, ,
2


  di chuyển về phía học sinh
n
lần lượt theo thứ tự
đó; các học sinh còn lại là
n
1
2,3,4, ,
2

di chuyển về phía học sinh 1 lần lượt theo thứ
tự đó để vào khoảng giữa hai học sinh này. Học sinh còn lại là
n
1
2

có thể di chuyển
theo hướng túy ý đến đúng vị trí của nó. Dễ thấy rằng: học sinh thứ
i

n i
1
 
với
n
i
1
2,3,4, ,
2

 đều cần số lần đổi chỗ là

i
1

. Do đó, tổng số lần đổi chỗ của các học
sinh này là
n n n n n
k
2
3 1 3 1 1
2 1 2 3
2 2 2 2 2
         
       
    
    
.
Từ đây ta có thể kết luận rằng số lần đổi chỗ nhỏ nhất cần sử dụng là
n n
( 2)
4
  
 
 
(biểu
thức này tương ứng bằng với các giá trị đã nêu trong từng trường hợp chẵn lẻ của
n
).
Bài toán được giải quyết hoàn toàn.
Bài 2.
Với mỗi số tự nhiên n, ta định nghĩa số

f n
( )
như sau:
f
(1) 1

và khi
n
1

thì
k k
f n a p a p a p
1 1 2 2
( ) 1      , trong đó
k
a
a a
k
n p p p
1 2
1 2

 là sự phân tích thành thừa số
nguyên tố của n (các số nguyên tố
k
p p p p
1 2 3
, , , ,
là đôi một khác nhau và

k
a a a a
1 2 3
, , , ,
là nguyên dương). Với mỗi số tự nhiên
k
, ta đặt




k
f n f f f n
( ) ( )

,
trong đó ở vế phải có đúng
k
lần chữ f . Chứng minh rằng với số tự nhiên
a
cho trước,
tồn tại số tự nhiên
k
0
để với mọi số nguyên
k k
0

thì tổng
k k

f a f a
1
( ) ( )

 không phụ
thuộc vào
k
.
Lời giải.
Trước hết, ta sẽ chứng minh các bổ đề sau:
11

(1) Với mọi số nguyên dương
n n
1 2
, ,
ta có f n n f n f n
1 2 1 2
( ) ( ) ( ) 1.
  

(2) Nếu
n
là hợp số và
n
9

thì
f n n
( ) 2.

 

(3) Nếu
n
là số nguyên tố và
n
9

thì
f n n
2
( ) .


(4) Với mỗi số tự nhiên
k
,
nếu
k
f a
( ) 9

thì tồn tại số tự nhiên
h k

thỏa
h k
f a f a
( ) ( ).


Chứng minh.
(1) Ta thấy hàm số đã cho có thể định nghĩa theo cách đơn giản hơn là:
f
(1) 1


m
f n p p p
1 2
( ) 1
    
trong đó
m
n p p p p
1 2 3

 (các số nguyên tố này
không nhất thiết phân biệt).
Do đó, giả sử
m
n p p p p
1 1 2 3

 và
n
n q q q q
2 1 2 3

 thì





m n
n n p p p p q q q q
1 2 1 2 3 1 2 3

 nên
m n
i i
i i
f n n p q f n f n
1 2 1 2
1 1
( ) 1 ( ) ( ) 1.
 
     
 

(2) Ta sẽ chứng minh bằng quy nạp. Thật vậy,
- Với
n
9

thì f f
2
(9) (3 ) 1 3 3 7 9 2
      
nên (2) đúng.
- Giả sử (2) đúng với với tất cả các hợp số

k n
9 ,
 
trong đó
n
là hợp số lớn hơn 9.
Ta đặt
n n n
1 2
 với
n n n
1 2
1 .
  
Ta thấy rằng n n
1 2
( 1)( 1) 4.
  

Thật vậy, nếu n
1
3

thì n n n n
2 1 1 2
1 1 2 ( 1)( 1) 4.
       

Nếu n
1

2

thì rõ ràng n
2
5

và ta cũng có n n
1 2
( 1)( 1) (5 1)(2 1) 4.
     

Tiếp theo, ta sẽ chứng minh rằng
i i
f n n
( ) 1
 
với mọi số nguyên dương
i
n
.
(*)
Thật vậy, nếu
i
n
là số nguyên tố thì rõ ràng
i i
f n n
( ) 1.
 


Nếu
i
n
là hợp số thì theo giả thiết quy nạp, ta có
i i i
f n n n
( ) 2 1.
   
Đồng thời
f f f
(4) 1 2 2 5 4 1, (6) 1 2 3 6 6 1, (8) 1 2 2 2 7 8 1
                  
.
Do đó, (*) đúng với mọi số nguyên dương
i
n
.
Suy ra
f n f n n f n f n n n n n
n n n n n
1 2 1 2 1 2 1 2
1 2
( ) ( ) ( ) ( ) 1 1 1 1 1
( 2) ( 1)( 1) 2 4 2.
            
         

(3) Nếu
n
là số nguyên tố lớn hơn 9 thì với

f n n
( ) 1
 
là một hợp số lớn hơn 9, theo bổ
đề (2) thì
f n f n n n n
2
( ) ( 1) 1 2 1 .
       

(4) Nếu
k
f a
( )
là hợp số thì theo bổ đề (2), ta có


k k k k
f a f f a f a f a
1
( ) ( ) ( ) 2 ( ).

   
Nếu
k
f a
( )
là số nguyên tố thì theo bổ đề (3), ta có



k k k
f a f f a f a
2 2
( ) ( ) ( ).

 
Từ hai trường hợp trên, ta thấy bổ đề (4) được chứng minh.
Trở lại bài toán đã cho,
12

Từ bổ đề (4), ta thấy rằng với mỗi giá trị
k

k
f a
( ) 9

thì đều tồn tại một dãy giảm
đến
k
f a
0
( ) 8.

Do đó, ta xét các trường hợp sau:
- Nếu
k
f a
0
( ) 8


thì dãy


k
f a
( )
với
k k
0


7,8,7,8,
nên
k k
f a f a k k
1 0
( ) ( ) 15, .

  
- Nếu
k
f a
0
( ) 7

thì dãy


k

f a
( )
với
k k
0


8,7,8,7,
nên
k k
f a f a k k
1 0
( ) ( ) 15, .

  
- Nếu
k
f a
0
( ) 6

thì dãy


k
f a
( )
với
k k
0



6,6,6,6,
nên
k k
f a f a k k
1 0
( ) ( ) 12, .

  

- Nếu
k
f a
0
( ) 5

thì
k
f a
0
1
( ) 6


nên dãy


k
f a

( )
với k k
0
1
 

6,6,6,6,
suy ra, tổng
k k
f a f a k k
1 0
( ) ( ) 12, 1.

   

- Nếu
k
f a
0
( ) 4

thì
k
f a
0
2
( ) 6


nên dãy



k
f a
( )
với k k
0
2
 

6,6,6,6,
suy ra, tổng
k k
f a f a k k
1 0
( ) ( ) 15, 2.

   

- Nếu
k
f a
0
( ) 3

thì
k
f a
0
3

( ) 6


nên dãy


k
f a
( )
với k k
0
3
 

6,6,6,6,
suy ra, tổng
k k
f a f a k k
1 0
( ) ( ) 15, 3.

   

- Nếu
k
f a
0
( ) 2

thì

k
f a
0
4
( ) 6


nên dãy


k
f a
( )
với k k
0
4
 

6,6,6,6,
suy ra, tổng
k k
f a f a k k
1 0
( ) ( ) 15, 4.

   

- Nếu
k
f a

0
( ) 4

thì
k
f a
0
2
( ) 6


nên dãy


k
f a
( )
với k k
0
2
 

6,6,6,6,
suy ra, tổng
k k
f a f a k k
1 0
( ) ( ) 15, .

  

- Nếu
k
f a
0
( ) 1

thì dãy


k
f a
( )
với
k k
0


1,1,1,1,
nên tổng
k k
f a f a k k
1 0
( ) ( ) 2, .

  
Vậy từ các trường hợp trên, ta thấy rằng với số tự nhiên
a
cho trước, tồn tại số tự nhiên
k
0

để với mọi số nguyên
k k
0

thì tổng
k k
f a f a
1
( ) ( )

 không phụ thuộc vào
k
.
Bài 3.
Trong một hội thảo khoa học, tất cả các đại biểu biết
n
2
ngôn ngữ,
n
2.

Mỗi người biết
đúng 2 ngôn ngữ và hai đại biểu bất kì biết chung nhiều nhất 1 ngôn ngữ. Biết rằng một
số nguyên
k
thỏa mãn
k n
1 1
  
đều có không quá

k
1

ngôn ngữ mà mỗi ngôn ngữ
này đều có không quá
k
người biết. Chứng minh rằng ta có thể chọn ra một nhóm
n
2

đại biểu biết tổng cộng
n
2
ngôn ngữ và mỗi ngôn ngữ có đúng 2 người trong đó biết.
Lời giải.
Ta xây dựng đồ thị
G V E
( , )

mà mỗi đỉnh biểu diễn một ngôn ngữ và cạnh nối hai
ngôn ngữ biểu diễn người biết hai ngôn ngữ đó. Ta có
V n
2
 và do mỗi người biết
chung không quá 1 ngôn ngữ nên
G
là đồ thị đơn.
13

Điều kiện còn lại là: với mỗi số nguyên

k

k n
1 1
  
thì có không quá
k
1

đỉnh
mà mỗi đỉnh có bậc không quá
k
.

Ta cần chứng minh rằng có thể chọn ra đúng
n
2
cạnh nối
n
2
đỉnh và mỗi đỉnh thuộc về
đúng 2 cạnh hay tồn tại một chu trình
H
có độ dài
n
2
đi qua tất cả các đỉnh của đồ thị.
Giả sử ngược lại là trong
G
không tồn tại chu trình

H
.
Khi đó, tập các đỉnh không kề
nhau của
G
là không rỗng (dễ thấy tập này cũng hữu hạn).
Ta thêm các cạnh vào đồ thị
G
để được đồ thị
G

thỏa mãn các điều kiện:
(1) Với mỗi số nguyên
k

k n
1 1
  
thì có không quá
k
1

đỉnh mà mỗi đỉnh có bậc
không quá
k
.

(2) Trong
G


không tồn tại chu trình nào.
(3) Nếu thêm bất kì cạnh nối 2 đỉnh không kề nhau của
G

thì đồ thị
G

sẽ có chu trình.
Rõ ràng với đồ thị xây dựng được như thế thì giữa hai đỉnh
u v
,
không kề nhau của đồ
thị
G

, luôn tồn tại một đường đi độ dài
n
2 1

qua tất cả các đỉnh của đồ thị và có đầu
mút là
u v
,
.
Với hai đỉnh
v v
,

của
G


, nếu
v n v n
deg( ) ,deg( )

 
thì
v v
,

phải kề nhau. Thật vậy, giả
sử hai đỉnh này không kề nhau thì tồn tại một đường đi
n
v v v v
1 2 3 2
, , , ,
đi qua tất cả các
đỉnh của
G

, trong đó
n
v v v v
1 2
,

 
.
Giả sử
v s n

deg( )
 
và gọi
s
i i i s
v v v i i i n
1 2
1 2
, , , (2 2
    
) là các đỉnh kề với
v v
1

.
Khi đó, các đỉnh
j s
1,2,3, ,

thì các đỉnh
j
i
v
1

đều không kề với
n
v v
2



vì nếu không
thì
j j
i n n i
v v v v v v v
1 2 1 2 2 1 1

 
là một chu trình của
G

, mâu thuẫn.
Từ đó suy ra
v n s n
deg( ) 2 ( 1) 1

    
, mâu thuẫn với
v n
deg( )


, do đó
v v
,

kề nhau.
Do đó, tập hợp
X

các đỉnh có bậc không vượt quá
n
1

của
G

là tập hợp không rỗng,
ta đặt
v X
v m n
maxdeg( ) 1.

  
Lấy đỉnh
v
1
trong tập hợp
X
sao cho
v m
1
deg( )

.
Từ điều kiện (1), ta có ít nhất
n n n
2 ( 1) 1 2
    
đỉnh có bậc lớn hơn hoặc bằng

n
. Suy
ra có đường đi
n
v v v
1 2 2
, , ,
qua tất cả các đỉnh của
G

, có độ dài
n
2 1

.
14

Đặt
m
i i i m
v v v i i i n
1 2
1 2
, , , (2 2
     ) là các đỉnh kề với
v
1
thì với
j n
1

 
, ta có
j
i
v
1


không kề với
n
v
2
chứng tỏ mọi đỉnh của
G

có bậc không nhỏ hơn 2. Xét điều kiện (1)
với
m
,
ta có


m
i i i
v v v
1 2
1 1 1
, , ,
  
phải chứa ít nhất một đỉnh

q
v

q
v m
deg( ) 1
 
.
Theo cách chọn giá trị
m
thì
q
v n
deg( )


q n
v v
2
,
có bậc không nhỏ hơn
n
nhưng lại
không kề nhau, mâu thuẫn.
Vậy ta có đpcm.
Bài 4.
Xét
n
điểm
n

A A A
1 2
, , ,
với
n
2

trong không gian, trong đó không có 4 điểm nào đồng
phẳng. Mỗi cặp điểm
i j
A A
,
với
i j

được nối với nhau bởi một đoạn thẳng.Tìm giá trị
lớn nhất của
n
sao cho có thể tô tất cả các đoạn thẳng đó bởi hai màu xanh hoặc đỏ
thỏa mãn đồng thời các điều kiện sau:
1) Mỗi đoạn thẳng được tô bằng đúng một màu.
2) Với mỗi
i n
1,2,3, ,

thì số đoạn thẳng có một đầu mút là
i
A
được tô màu xanh
không vượt quá 4.

3) Với mỗi đoạn thẳng
i j
A A
được tô màu đỏ đều tìm được ít nhất một điểm
k
A
với
k

khác
i j
,
sao cho các đoạn thẳng
i k j k
A A A A
, được tô màu xanh.
Lời giải.
Xét
n
điểm
n
A A A
1 2
, , ,
mà có thể tô màu các đoạn
i j
A A
thỏa mãn đề bài. Xét graph
G V E
( , )


với


n
V A A A
1 2
, , , và tập cạnh được tô màu xanh. Dễ thấy
G
là đồ thị
đơn, vô hướng và thỏa mãn các điều kiện:
i)
i
A i n
deg( ) 4, 1, .
 
ii) Với bất kì hai đỉnh
i j
A A
,
nào của đồ thị thì đều tồn tại một xích đơn nối chúng có độ
dài không vượt quá 2.
Ta cần tìm giá trị lớn nhất của
n
.

15

Xét một đỉnh
i

A
tùy ý, ta thấy rằng với
j
A
là một trong
n
1

đỉnh còn lại thì
j
A
kề với
i
A
hoặc kề với một cạnh kề với
i
A
. Từ điều kiện i), suy ra
n
1 4 3 4 17.
    

Ta xét các trường hợp sau:
- Nếu
n
17

thì ta phải có
i
A i n

deg( ) 4, 1,
  và E
4 17
34
2

 
. Xét đỉnh
i
A
bất kì của
đồ thị này và các đỉnh
i i i i
A A A A
1 2 3 4
, , ,
. Gọi các đỉnh còn lại của
G
, khác
i
A
là đỉnh rìa
và cạnh nối hai đỉnh rìa là cạnh rìa.
cạnh rìa
A
i

Do đó, không có hai đỉnh nào của
G
cùng với

i
A
lập thành ba đỉnh đôi một kề nhau và
như thế, với mỗi cạnh rìa, ta có một chu trình đơn có độ dài 5 qua
i
A
.
Số cạnh rìa là
34 4 4 3 18
   
nên có tất cả 18 chu trình đơn độ dài 5 qua
i
A
. Vì
i
A

đỉnh tùy ý nên số chu trình đơn độ dài 5 của
G
có tất cả là
18 17
5



, vô lí.
Suy ra
n
17


không thỏa mãn.
- Nếu
n
16

thì ta cũng phải có
i
A i n
deg( ) 4, 1,
  và E
4 16
32
2

 
. Xét một đỉnh
i
A

nào đó của
G
.
Tương tự như trên, ta giả sử
i
A
kề với 4 đỉnh rìa là
i i i i
A A A A
1 2 3 4
, , ,

và có
đúng 1 đỉnh rìa, giả sử là
k
A
kề với 2 đỉnh rìa khác. Dễ thấy trong
G
không có 3 đỉnh
nào đôi một kề nhau nên với mỗi cạnh rìa liên thuộc
k
A
cho ta 2 chu trình đơn độ dài 5
qua
i
A
và mỗi cạnh rìa không liên thuộc
k
A
cho ta 1 chu trình đơn độ dài 5 qua
i
A
.

16

A
i
A
k

Số cạnh rìa là

32 16 16
 
và trong đó, có đúng 2 cạnh liên thuộc
k
A
.
Các chu trình này
đôi một phân biệt nên tổng số chu trình như thế là
14 1 2 2 18
   
và tổng số chu trình
đơn độ dài 5 của đồ thị là
18 16
5



, vô lí.
Suy ra
n
16

cũng không thỏa mãn.
- Nếu
n
15,

ta xây dựng đồ thị như sau
A
7

A
8
A
9
A
10
A
12
A
13
A
14
A
15
A
1
A
6
A
11
A
2
A
3
A
4
A
5

Ta thấy đồ thị này thỏa mãn tất cả yêu cầu của đề bài.

Vậy giá trị lớn nhất của
n
cần tìm là 15.
17

Bài 5.
Xét phương trình nghiệm nguyên dương
x y z t Nxyzt N
2 2 2 2
0
     

với
N
là số nguyên dương cho trước.
1) Chứng minh rằng tồn tại vô số giá trị
N
sao cho phương trình này có nghiệm nguyên
dương (mỗi nghiệm là một bộ bốn số nguyên dương
x y z t
, , ,
).
2) Chứng minh rằng với
k
N m
4 (8 7)
 
với
k m
,

là các số không âm thì phương trình
này không có nghiệm nguyên dương.
Lời giải.
1) Viết lại phương trình đã cho dưới dạng
t t Nxyzt N x y z
2 2 2
( ) ( )
     . (1)
Khi đó, nếu
N a b c
2 2 2
  
với a b c, ,



thì phương trình trên có nghiệm là
x a y b z c t Nabc a b c abc
2 2 2
0 0 0 0
, , , ( )
       . (*)
Từ đây suy ra tồn tại vô số giá trị
N
để phương trình đã cho có nghiệm nguyên dương.
2) Giả sử với
k
N m
4 (8 7)
 

, phương trình đã cho có nghiệm
x y z t
0 0 0 0
( , , , )
sao cho tổng
x y z t
0 0 0 0
  
nhỏ nhất. Do tính bình đẳng giữa các biến, ta giả sử
x y z t
0 0 0 0
  
. Ta
sẽ chứng minh rằng với
N
7

, nếu (1) có nghiệm thì các nghiệm phải có dạng (*).
Ta thấy
t
0
là nghiệm của phương trình bậc hai t Nx y z t x y z N
2 2 2 2
0 0 0 0 0 0
( ) 0
      
.
Theo định lí Viete thì phương trình này sẽ có một nghiệm
t
1

thỏa mãn
t t Nx y z
t t x y z N
0 1 0 0 0
2 2 2
0 1 0 0 0
( )
  


   



Suy ra t
1


, ta cũng có N x y z t t x y z t t
x y z
2 2 2 2
0 0 0 1 1 0 0 0 1 1
0 0 0
1
(1 ) 0 1 0
            
.
Giả sử t
1
0


thì
x y z t
0 0 0 1
( , , , )
cũng là nghiệm nguyên dương của phương trình (1). Do
cách chọn bộ
x y z t
0 0 0 0
( , , , )
nên
t t
1 0

, suy ra
18

t t t x y z N x y z z
2 2 2 2 2 2 2 2
0 1 0 0 0 0 0 0 0 0
3
         nên
N x y z N x y z t x y z t z Nz
2 2 2 2 2 2 2
0 0 0 0 0 0 0 0 0 0 0 0 0
(1 ) (1 ) 6         (do
N
7

)

Ta đây ta có
x y z z
2 2
0 0 0 0
1
 
, vô lí. Do đó t
1
0

hay
N x y z t Nx y z
2 2 2
0 0 0 0 0 0 0
,    , nghĩa là
nếu phương trình
(1)
có nghiệm thì
N
phải có dạng như thế.
Ta sẽ chứng minh rằng với
k
N m
4 (8 7) 7
  
thì phương trình
N x y z
2 2 2
  
không

có nghiệm nguyên dương.
Ta xét các trường hợp sau:
- Nếu
k
0

thì có phương trình x y z m
2 2 2
8 7
   
hay x y z
2 2 2
7(mod8).
  
Suy ra trong các số
x y z
, ,
phải có một số lẻ hoặc cả ba số lẻ. Tuy nhiên, ta cũng thấy
rằng


x y z
2 2 2
, , 0,1,4
 nên rõ ràng phương trình trên không có nghiệm nguyên dương.
- Nếu
k
0

thì x y z

2 2 2
0(mod4)
   nên trong các số
x y z
, ,
phải có ba số chẵn hoặc
một số chẵn. Nếu có một số chẵn, hai số lẻ thì x y z
2 2 2
2(mod4)
   , không thỏa. Còn
nếu cả ba số đều chẵn thì đặt
x x y y z z
1 1 1
2 , 2 , 2
   đưa về
k
x y z m
2 2 2 1
1 1 1
4 (8 7)

   
,
sau
k
lần biến đổi tương tự thì đưa về phương trình trong trường hợp trên và như đã
chứng minh thì nó cũng vô nghiệm.
Vậy trong mọi trường hợp thì phương trình
k
x y z m

2 2 2
4 (8 7)
   
không có nghiệm
nguyên dương, dẫn đến phương trình đã cho cũng không có nghiệm nguyên dương.
Ta có đpcm.
Bài 6.
Cho hàm số thực
x
f x
x
3
2
2 3
( )
3( 1)



.
1. Chứng minh rằng tồn tại hàm số
g x
( )
liên tục trên

sao cho
f g x x
( ( ))



g x x
( )


với mọi số thực x.
2. Chứng minh rằng tồn tại số thực
a
1

để dãy
n
u n
( ), 1,2,3,
 được xác định bởi
n n
u a u f u n
0 1
, ( ), 1,2,3,

   tuần hoàn với chu kỳ dương nhỏ nhất bằng 1995.
19

Lời giải.
1. Hàm số
f x
( )
xác định trên
( ; 1) ( 1;1) (1; )
     
.

Ta có
x x x
f x
x
3
2 2
2 ( 3 3)
( )
3( 1)
 



.
Ta thấy rằng x x x x x
3 2
3 3 ( 1) ( 2) 1 0, 1
        
nên hàm số đã cho đồng biến trên
(1; )

.
Hơn nữa,
x
x
f x f x
1
lim ( ) , lim ( )




   
nên với tập xác định là
(1; )

thì tập giá trị
của
f x
( )

( ; )
 
, mà
f x
( )
là hàm liên tục và đồng biến trên
(1; )

nên nó chính là
một song ánh từ
(1; )

vào
( ; )
 
. Do đó, tồn tại hàm số ngược
g x
( )
của
f x

( )

tập xác định là

và tập giá trị là
(1; )

sao cho
f g x x x
( ( )) ,
  

.
Ta cần chứng minh rằng
g x x x
( ) ,
  

. Thật vậy:
- Với
x
1

thì
g x x
( ) 1
 
.
- Với
x

1

thì
x x
f x x x x f x x f g x f x x
x
3
2
3 3
( ) 0, 1 ( ), 1 ( ( )) ( ), 1
3( 1)
 
             

,
do tính đồng biến của
f x
( )
trên
(1; )

nên
g x x x
( ) , 1
  
.
Vậy hàm số
g x
( )
thỏa mãn đề bài.

Ta có đpcm.
2. Đặt
n
n
g x g g g x n
*
( ) ( ( )) ), 


và u a g x
0 1994 0
( ) 1
  
với x
0


là tham số sẽ được
xác định sau.
Ta có u g x u f u f g x g x
0 1994 0 1 0 1994 0 1993 0
( ) 1 ( ) ( ( )) ( ) 1
      
. Tiếp tục quá trình này, ta
dễ dàng chứng minh được
u x
1994 0

đồng thời vì
g x x x

( ) ,
  

nên
u u u u
0 1 2 1994
    .
Do đó, nếu chọn được
x
0
sao cho x
0
1
 

n
u f x g x
1995 0 0
( ) ( )
  thì dãy đã cho có chu
kì dương nhỏ nhất là 1995. Ta chỉ cần chứng minh rằng tồn tại
x
0
thỏa mãn điều kiện
nêu trên.
Thật vậy, xét hàm
n
h x f x g x
( ) ( ) ( )
  trên

( 1;0]

, dễ thấy
h x
( )
là hàm liên tục trên nửa
khoảng này và
20

n n
h f g g
(0) (0) (0) 1 (0) 0
    
,
n
x x
h x f x g
1 1
lim ( ) lim ( ) ( 1)
 
 
    

nên phương trình
h x
( ) 0

có nghiệm
x
0

trên
( 1;0]

.
Vậy tồn tại giá trị
a
1

thỏa mãn đề bài nên ta có đpcm.
Bài 7.
Xét các số thực
a b c
, , .
Tìm giá trị nhỏ nhất của biểu thức
f a b c a b b c c a a b c
4 4 4 4 4 4
4
( , , ) ( ) ( ) ( ) ( )
7
         .
Lời giải.
Ta sẽ chứng minh rằng
f a b c
( , , ) 0

với mọi
a b c
, ,



, tức là
a b b c c a a b c
4 4 4 4 4 4
4
( ) ( ) ( ) ( )
7
        .
Ta có bất đẳng thức mạnh hơn như sau
 
a b b c c a a b c a b c
4 4 4 4 4 4 4
4
( ) ( ) ( ) ( )
7
           . (*)
Đặt
y z x z x y x y z
a b c, ,
2 2 2
     
   , dễ thấy các số
x y z
, ,
luôn tồn tại. Thay vào
bất đẳng thức (*), chú ý rằng
a b z b c x c a y
, ,
     

x y z

a b c
2
 
   .
y z x z x y x y z x y z
x y z
4 4 4 2
4 4 4
4
7 2 2 2 2
 
               
 
     
       
 
       
 

x y z x y z y z x z x y x y z
4 4 4 4 4 4 4
28( ) ( ) ( ) ( ) ( )
              
(**)
Với mọi
m n
,
thì
m n m n m n m n
4 4 4 4 2 2

( ) ( ) 2( 6 )
      . Suy ra


x y z y z x y z x y z x
4 4 4 4 2 2
( ) ( ) 2 ( ) 6( )         



z x y x y z x y z x y z
4 4 4 4 2 2
( ) ( ) 2 ( ) 6 ( )
         

21

Bất đẳng thức (**) có thể viết lại là


x y z x y z y z x y z y z
x y z x y z y z x y z
x y z x y z y z x y z
x y z y z x y z
x y z x y y z z x
x y y z
4 4 4 4 4 4 2 2 2
4 4 4 4 4 4 2 2 2 2 2
4 4 4 4 4 4 2 2 2 2 2
4 4 4 2 2 2 2 2

4 4 4 2 2 2 2 2 2
2 2 2 2 2
14( ) 2 ( ) ( ) 6 ( ) ( )
14( ) 2 2( 6 ) 24 ( )
7( ) 6 12 ( )
6( ) 6 12 ( )
( ) (
          
        
        
     
     
    z x
2 2 2 2
) ( ) 0  

Bất đẳng thức cuối này đúng nên bất đẳng thức (*) đúng và bất đẳng thức đã cho được
chứng minh.
Đẳng thức xảy ra chẳng hạn tại
a b c
0
  
.
Bài 8.
Tìm số thực
k
lớn nhất sao cho tồn tại vô hạn số tự nhiên
n
a n
( ), 1


thỏa mãn
1)
n
n
a
1997
 với mọi
n
.

2) Với mỗi
n
2,

ta đều có
k
n n
u a

với
n
u
là ước chung lớn nhất của tất cả các số có dạng
i k
a a


i k n
.

 

Lời giải.
Ta thấy rằng
k
0

thỏa mãn điều kiện của đề bài nên để tìm
k
lớn nhất, ta xét
k
0.


Từ điều kiện 2, ta thấy rằng
k
i j i j
a a a i j
*
, ,

   

, suy ra
i i
k
a a i
1
2 2
2 ,


  

. (*)
Ta xét các trường hợp sau:
- Nếu
k
1

thì bất đẳng thức (*) viết lại là
i i
k
k k
a a a
1
1 1
1 1
2 2
2

 
 

 
 
hoặc
i i
k
k k
a a a

1/
1 1
1 1
2 2
2
 
 

 
 
.
Suy ra
n n n
k kn
k k k k
a a a a a a n
1
1/ 1/
1 1 1 1
*
1 1 1 1
1
2 2 2
2 2 ,

   
   
    
   
   


.
Hơn nữa,
n
n
n
n
a n a
* 2
2
1997 , 1997
     nên
n
kn
k k
a
1/
1 1
2
1 1
1
2 1997 2
 
 
 
 
 
. Do đó,
n
k k

n
k k
n n
a n
k
a
k
1 1
*
1 1
1
1 1
1 1
1
1
ln2 2 ln1997 ln 2 ,
1 1
ln1997 ln 2 ln2
(2 ) 2
 
 
 
   
 
 
 
  
 
 



22

Từ đây suy ra k
1
2

vì nếu không thì khi
n
 
, bất đẳng thức trên sẽ không đúng.
- Nếu
k
1

thì ta có
i i
a a
1
2 2
2

 hay
n
n n
n
a a a a n
1
2 *
1

2 2
2 2 ,

    

, mà
n
n
a
2
2
1997
 nên
n
n
a
2
1
1997 2
 , suy ra
n n
a
n
n
*
1
ln
ln1997 ln2 ,
2 2
   


.
Cho
n
,
 
ta thấy bất đẳng thức trên không đúng nữa.
Do đó, ta luôn có k
1
2

. Ta sẽ chứng minh rằng k
1
2

thỏa mãn đề bài. Thật vậy,
Xét dãy số
n
a
( )
như sau
n n n
a a
a a a n
2
1 2
1 1
2 1997, 4 1997 2
2 1997 , 2
 


    


   


.
Công thức tổng quát
n
a
( )

n n
n
a p q
 
với p q
2 2
1997 1997 1, 1997 1997 1
     
.
Ta có
n
n n n n
n
p q
a p q n
*
2 2 1997 1997 ,

2
  
       
 
 

.
Tiếp theo, ta sẽ chứng minh rằng
n
n n
u a a n
1
2 2
, 2
    
   
   
   
(chú ý
n n
n
1
2 2
    
 
   
   
) hay
n n n n
t t

n
a a a a n t
1 1
2 2 2 2
, 2, 0,1,2, , 1
2
         
 
       
       
   
 
 
   
     
 
 
   
 
 
   
 (*)
bằng quy nạp theo
t
.

- Với
t
0


thì (*) hiển nhiên đúng.
- Với
t
1

thì ta có 2 trường hợp:
+ Nếu n m m
*
2 , 

, ta có


m m m m m
n n
a a a a a a a
1 1
1
1 1
2 2
2 1997
 
    
 
   
   
      hay
n n n n
a a a a
1 1

1 1
2 2 2 2
         
 
       
       
   
   
 
   
   
 .
+ Nếu n m m
*
2 1,  

, ta có
m m m m m m
n n
m m m m
a a a a a a a a
a a a a
1 2 1 1
1
1 1
2 2
1 1
2 1997 2 1997
(2 1997 1)( ) ( )
   

    
 
   
   
 
         
    

Do đó, (*) đúng với
t
1,

ta sẽ chứng minh nếu (*) đúng với
n
t t
,1 1
2
 
  
 
 
thì (*) cũng
đúng với
t
1

. Thật vậy, ta có
23

n n n n n n

t t t t t t
n n n n
t t t t
a a a a a a
a a a a
1 1 1
( 1) ( 1) 1 1
2 2 2 2 2 2
1 1
( 1) ( 1)
2 2 2 2
2 1997 2 1997
2 1997
              
         
           
           
         
     
       
       
      
   
   
    
   
   

Theo giả thiết quy nạp thì ta thấy (*) đúng với
t

1.


Do đó, (*) đúng với mọi
n
2

.
Từ đây, ta suy ra
   
n n n n
n n n n
n n n
u p q p q p q p q a a
1 1
1/2
2 2 2 2
/2 /2 1/2 1/2
2 2 2
         
       
       
          .
Vì thế, dãy số
n
a
( )
được xây dựng như trên thỏa mãn tất cả các điều kiện của đề bài ứng
với k
1

.
2


Vậy giá trị lớn nhất cần tìm của
k

1
.
2

Bài 9.
Giả sử
d
là ước dương của
1998
5 1998

. Chứng minh rằng
d
có thể biểu diễn được dưới
dạng
d x xy y
2 2
2 2 3
   với
x y
,
là các số nguyên dương khi và chỉ khi
d

chia 20 có số
dư là 3 hoặc 7.
Lời giải.
Điều kiện cần.
Ta giả sử
d x xy y
2 2
2 2 3
   với
x y
,
là các số nguyên dương nào đó.
Đặt
N
1998
5 1998
 
, ta thấy
N
lẻ, không chia hết cho 5 nên
d
cũng lẻ và không chia hết
cho 5. Ta có
d x x y y
2
2 ( ) 3
   , dẫn đến
y
lẻ.
Suy ra với mọi

x
thì
x x y
2 ( )

đều chia hết cho 4.
Do đó, ta luôn có d y
2
3 3(mod 4)
  và d x y y x y
2 2 2
2 (2 ) 5 (2 ) (mod 5)
     , suy ra
d
2 1(mod5)
 
hay
d
2(mod5)
 
, tức là
d
chia 5 dư 2 hoặc dư 3.
- Nếu
d
chia 5 dư 2 và chia 4 dư 3 thì
d
chia 20 dư 7.
- Nếu
d

chia 5 dư 3 và chia 4 dư 3 thì
d
chia 20 dư 3.
Do đó, điều kiện cần được chứng minh.
24

Điều kiện đủ.
Đặt
a
999
1998

thì
N a
2
5
 
. Xét tập hợp


A ax y x d y d
| 0,1,2, , , 0,1,2,3, ,
   
   
   
.
Đặt
q d
 


 
thì q d q
1.
  
Dễ thấy
A q d
2
( 1)
  
nên tồn tại
x y x y
1 1 2 2
( ; ),( ; )
để
ax y ax y d
1 1 2 2
(mod )
   hay
a x x y y d
1 2 1 2
( ) 0(mod )
    .
Đặt
x x x y y y
0 1 2 0 1 2
,
   
thì
ax y d
0 0

0(mod )
  , do đó
a x y ax y ax y d
2 2 2
0 0 0 0 0 0
( )( ) 0(mod )
     .
Mặt khác
a d
2
5(mod )
  nên
x y d x y d
2 2 2 2
0 0 0 0
5 0(mod ) 5    

.
Ta cũng có
x x x q d y y y q d
0 1 2 0 1 2
,        nên
x d y d
2 2
0 0
,
 

x y d
2 2

0 0
5 6
 
.
Từ đây, ta suy ra
x y kd
2 2
0 0
5
 
với
k
0,1,2,3,4,5,6

.
Ta xét từng trường hợp:
- Nếu x y x y
2 2
0 0 0 0
5 0 0
    
hay
x x y y
1 2 1 2
,
 
, vô lí.
- Nếu
x y d
2 2

0 0
5
 
thì y d
2
0
(mod 5)
 và y
2
0
2(mod5
  ), vô lí.
- Nếu
x y d
2 2
0 0
5 2
  thì x y d
2 2
0 0
2 (mod4)
  , mà
d
3(mod4)

nên x y
2 2
0 0
2(mod4)
  và

do đó,
x y
0 0
,
cùng lẻ. Đặt y x y y
0 0
2 ,
  

thì ta được
x y x x y d x x y y d y x y x d
2 2 2 2 2 2 2 2
0 0 0 0 0 0 0 0
5 5 ( 2 ) 2 6 4 4 2 2 2 3
            

Suy ra
d
biểu diễn được dưới dạng
x xy y
2 2
2 2 3
  , thỏa mãn đề bài.
- Nếu
x y d
2 2
0 0
5 3
 
thì y x y x y x

2 2
0 0 0 0 0 0
0(mod 3) ( )( ) 0(mod 3)
      .
+ Nếu
y x
0 0

chia hết cho 3 thì đặt y x y y
0 0
3 ,
  

và ta được
x y x x y x x y y d x x y y d
2 2 2 2 2 2 2 2
0 0 0 0 0 0 0 0
5 5 ( 3 ) 6 6 9 3 2 2 3
           
.
+ Nếu
y x
0 0

chia hết cho 3 thì đặt x y y y
0 0
3 ,
  

và ta được

x y x y x x yx y d x y x y d
2 2 2 2 2 2 2 2
0 0 0 0 0 0 0 0 0
5 5 (3 ) 6 6 9 3 2 2( ) 3( )
             
.
25

Trong cả hai trường hợp này
d
đều biểu diễn được dưới dạng
x xy y
2 2
2 2 3
  .
- Nếu
x y d
2 2
0 0
5 4
  thì y d
2
0
4 (mod 5)
 , dẫn đến y d
2
0
(mod5)
  và y
2

0
2(mod5
  ), vô lí.
- Nếu
x y d
2 2
0 0
5 5
 
thì
y y
0 1
5
 và
x y d
2 2
0 1
5
 
, suy ra x d
2
0
2(mod5)
   , vô lí.
- Nếu
x y d
2 2
0 0
5 6
 

thì y d
2
0
(mod 5)
 và cũng đưa về y
2
0
2(mod5
  , vô lí.
Do đó, điều kiện đủ được chứng minh.
Vậy bài toán được giải quyết hoàn toàn.
Bài 10.
Cho một số nguyên tố lẻ p thỏa mãn
h
p
2 1(mod )
 với mọi h p h
*
1,
  

và một số
chẵn
p
a p
( ; )
2
 . Xét dãy số
n
a

( )
xác định bởi
n n
a a a p b n
0 1
, , 0,1,2,

    với
n
b
là ước số lẻ lớn nhất của
n
a
.
Chứng minh rằng
n
a
( )
là dãy số tuần hoàn và tìm chu kì dương nhỏ nhất của nó.
Lời giải.
Ta thấy rằng với mọi i,
i
a
là số chẵn,
i i
a p b p
1
  

i

i i i
a
p p
b i a p b
1
1 1
,
2 2 2

 
      
.
Suy ra
i
p
a p i
,
2
  
, tức là giá trị của các số hạng của dãy
n
a
( )
đã cho là hữu hạn.
Đặt
i
k
i i
a b
2

 
với
i i i
k b k
, , 0
 


i
b
là số lẻ. Nếu
i j
b b

thì
i j
a a
|
hoặc
j i
a a
|
, theo
nhận xét trên thì
i j
a a

vì nếu ngược lại thì cả hai số này không thể nằm trong
p
p

,
2
 
 
 
.
Cũng theo nhận xét trên thì phải tồn tại các giá trị
p
i j i j p
; , ,
2
 
 
 
 
thỏa mãn
i j
a a

;
mặt khác
i i j j
a p b a p b
1 1
,
 
    nên
i j
b b


, do đó
i j
a a
1 1
 
 , tiếp tục quá trình này, ta
được
j i
a a
0

 , tức là dãy đã cho tuần hoàn từ số hạng đầu tiên. Nếu đặt
T j i
 
thì T
chính là chu kì dương nhỏ nhất cần tìm của dãy đã cho.

×